Tag Archives: 2012

ĐỀ THI VÀO CHUYÊN TOÁN LỚP 10 TP.HCM 2012

Bài 1. Giải phương trình:

$\sqrt{8 x+1}+\sqrt{46-10 x}=-x^{3}+5 x^{2}+4 x+1$

Bài 2. Cho đa thức $f(x)=a x^{3}+b x^{2}+c x+d$ với $a$ là số nguyên dương, biết $f(5)-$ $f(4)=$ 2012. Chứng minh rằng: $f(7)-f(2)$ là hợp số.

Bài 3. Cho ba số dương $a, b, c$ thỏa $a+b+c=1$. Tìm giá trị nhỏ nhất của biểu thức:

$A=14\left(a^{2}+b^{2}+c^{2}\right)+\frac{a b+b c+c a}{a^{2} b+b^{2} c+c^{2} a}$

Bài 4. Cho tứ giác $A B C D$ nội tiếp đường tròn $(O, R)$ có $A C$ vuông góc với $B D$ tại $H$. Trên cạnh $A B$ lấy điểm $M$ sao cho $A B=3 A M$. Trên cạnh $H C$ lấy trung điểm $N$. Chứng minh rằng $M H$ vuông góc với $D N$.

Bài 5. Cho đường tròn tâm $O$ và đường tròn tâm $I$ cắt nhau tại hai điểm $A$ và $B(O$ và $I$ nằm khác phía đối với đường thẳng $A B), I B$ cắt $(O)$ tại $E, O B$ cắt $(I)$ tại $F$. Qua $B$ vẽ đường thẳng $M N$ song song với $E F(M$ thuộc $(O), N$ thuộc $(I)$ ).

(a) Chứng minh rằng $O A I E$ nội tiếp.

(b) Chứng minh rằng: $A E+A F=M N$.

Bài 6. Trên mặt phẳng cho 2013 điểm tùy ý sao cho trong ba điểm bất kì thì tồn tại 2 điểm mà khoảng cách giữa hai điểm đó luôn bé hơn 1. Chứng minh rằng tồn tại một đường tròn có bán kính bằng 1 chứa ít nhất 1007 điểm (kế cả biên).

 

LỜI GIẢI

Bài 1. Giải phương trình:

$\sqrt{8 x+1}+\sqrt{46-10 x}=-x^{3}+5 x^{2}+4 x+1$

Lời giải. $\sqrt{8 x+1}+\sqrt{46-10 x}=-x^{3}+5 x^{2}+4 x+1$

ĐKХĐ: $\frac{-1}{8} \leq x \leq \frac{23}{5}$

Sử dụng lượng liên hợp, phương trình ban đầu tương đương với:

$\sqrt{8 x+1}-3+\sqrt{46-10 x}-6+x^{3}-x^{2}-4 x^{2}+4 x-8 x+8=0$

$\Leftrightarrow(x-1)\left(\frac{8}{\sqrt{8 x+1}+3}-\frac{10}{\sqrt{46-10 x}+6}+x^{2}-4 x-8\right)=0$

Từ đó ta có phương trình có một nghiệm là $x=1$. Xét biểu thức:

$\frac{8}{\sqrt{8 x+1}+3}-\frac{10}{\sqrt{46-10 x}+6}+x^{2}-4 x-8=0$

Từ điều kiện ta có:

$-1<x<5 \Leftrightarrow(x+1)(x-5)<0 \Leftrightarrow x^{2}-4 x-5<0$

Lại có: $\frac{8}{\sqrt{8 x+1}+3} \leq \frac{8}{3}<\frac{9}{3}=3 \Leftrightarrow \frac{8}{\sqrt{8 x+1}+3}-3<0$ Từ đó ta có:

$\frac{8}{\sqrt{8 x+1}+3}-\frac{10}{\sqrt{46-10 x}+6}+x^{2}-4 x-8<0$

Vậy phương trình đã cho có nghiệm duy nhất là: $x=1$

Bài 2. Cho đa thức $f(x)=a x^{3}+b x^{2}+c x+d$ với $a$ là số nguyên dương, biết $f(5)-$ $f(4)=2012$. Chứng minh rằng: $f(7)-f(2)$ là hợp số.

Lời giải. Ta có: $f(x)=a x^{3}+b x^{2}+c x+d$

Từ đó ta tính được: $f(5)=125 a+25 b+5 c+d, f(4)=64 a+16 b+4 c+d$

Vậy: $f(5)-f(4)=61 a+9 b+c=2012, f(7)=343 a+49 b+7 c+d, f(2)=8 a+4 b+$ $2 c+d$

Vậy: $f(7)-f(2)=335 a+45 b+5 c=5(67 a+9 b+c)=30 a+5(61 a+9 b+c)=30 a+$ 10060

Từ đó ta có: $f(7)-f(2)$ là hợp số vì $a$ là số nguyên dương và nó chia hết cho $2,5,10$.

Bài 3. Cho ba số dương $a, b, c$ thỏa $a+b+c=1$. Tìm giá trị nhỏ nhât của biểu thức:

$A=14\left(a^{2}+b^{2}+c^{2}\right)+\frac{a b+b c+c a}{a^{2} b+b^{2} c+c^{2} a}$

Lời giải.

Cách 1:

$\left(a^{2}+b^{2}+c^{2}\right)(a+b+c)=a^{3}+b^{3}+c^{3}+\left(a^{2} b+b^{2} c+c^{2} a\right)+\left(b^{2} a+a^{2} c+c^{2} b\right) $

$\left(a^{2}+b^{2}+c^{2}\right)(a+b+c)=\left(a^{3}+a b^{2}\right)+\left(b^{3}+b c^{2}\right)+\left(c^{3}+c a^{2}\right)+\left(a^{2} b+b^{2} c+c^{2} a\right)$

Áp dụng bất đẳng thức Cauchy và do $a+b+c=1$, ta có:

$\left(a^{2}+b^{2}+c^{2}\right) \geq 2 a^{2} b+2 b^{2} c+2 c^{2} a+\left(a^{2} b+b^{2} c+c^{2} a\right)=3\left(a^{2} b+b^{2} c+c^{2} a\right)$

Mặt khác: $a b+b c+c a=\frac{1-\left(a^{2}+b^{2}+c^{2}\right)}{2}$

Từ đó ta có: $F \geq 14\left(a^{2}+b^{2}+c^{2}\right)+\frac{3-3\left(a^{2}+b^{2}+c^{2}\right)}{2\left(a^{2}+b^{2}+c^{2}\right)}$

Hay: $F \geq 14\left(a^{2}+b^{2}+c^{2}\right)+\frac{3}{2\left(a^{2}+b^{2}+c^{2}\right)}-\frac{3}{2}$

Áp dụng bất đẳng thức Cauchy, ta có:

$27\left(a^{2}+b^{2}+c^{2}\right)+\frac{3}{\left(a^{2}+b^{2}+c^{2}\right)} \geq 2 \sqrt{27\left(a^{2}+b^{2}+c^{2}\right) \cdot \frac{3}{\left(a^{2}+b^{2}+c^{2}\right)}}=18 $

$a^{2}+b^{2}+c^{2} \geq \frac{1}{3}(a+b+c)^{2}=\frac{1}{3}$

Vậy: $28\left(a^{2}+b^{2}+c^{2}\right)+\frac{3}{\left(a^{2}+b^{2}+c^{2}\right)} \geq 18+\frac{1}{3}=\frac{55}{3}$

Từ đó ta có: $F \geq \frac{55}{6}-\frac{3}{2}=\frac{23}{3}$

Đẳng thức xảy ra khi: $a=b=c=\frac{1}{3}$

Cách 2:

Do $a, b, c$ dương và $a+b+c=1$ nên ta có:

$(1-c)^{2}=(a+b)^{2} \geq 4 a b \Leftrightarrow 1-2 c+c^{2} \geq 4 a b \Leftrightarrow a-2 a c+a c^{2} \geq 4 a^{2} b $

$(1-a)^{2}=(b+c)^{2} \geq 4 b c \Leftrightarrow 1-2 a+a^{2} \geq 4 b c \Leftrightarrow b-2 a b+a^{2} b \geq 4 b^{2} c $

$(1-b)^{2}=(c+a)^{2} \geq 4 c a \Leftrightarrow 1-2 b+b^{2} \geq 4 c a \Leftrightarrow c-2 b c+b^{2} c \geq 4 a c^{2}$

Hay: $a+b+c-2(a b+b c+c a) \geq 3\left(a^{2} b+b^{2} c+c^{2} a\right)$

$\Leftrightarrow 1-2(a b+b c+c a) \geq 3\left(a^{2} b+b^{2} c+c^{2} a\right)$

Vậy: $F \geq 14[1-2(a b+b c+c a)]+\frac{3(a b+b c+c a)}{1-2(a b+b c+c a)}$

Đạt: $t=1-2(a b+b c+c a), t \geq \frac{1}{3}$

Áp dụng bất đẳng thức Cauchy ta có:

$F \geq 14 t+\frac{\frac{3}{2}(1-t)}{t}=14 t+\frac{3}{2 t}-\frac{3}{2}=\frac{1}{2} t+\frac{27}{2} t+\frac{3}{2 t}-\frac{3}{2} \geq \frac{1}{2} t+2 \sqrt{\frac{27}{2} t \cdot \frac{3}{2 t}}-\frac{3}{2}$

Vậy: $F \geq \frac{1}{2} \cdot \frac{1}{3}+9-\frac{3}{2}=\frac{23}{3}$

Đẳng thức xảy ra khi: $a=b=c=\frac{1}{3}$

Bài 4. Cho tứ giác $A B C D$ nội tiếp đường tròn $(O, R)$ có $A C$ vuông góc với $B D$ tại $H$. Trên cạnh $A B$ lấy điểm $M$ sao cho $A B=3 A M$. Trên cạnh $H C$ lấy trung điểm $N$. Chứng minh rằng $M H$ vuông góc với $D N$.

Lời giải.

  • Gọi $K, L$ lần lượt là trung điểm $B M$ và $H B, P$ là giao điểm của $H M$ và $A K$.

  • Ta có $K L$ là đường trung bình của tam giác $H M B$ nên $K L$ song song $H M$. Khi đó xét tam giác $A K L$ thì $P H$ là đường trung bình nên $P$ là trung điểm của $A K$.

  • Ta có từ $A B C D$ nội tiếp suy ra $H D \cdot H B=H A \cdot A C \Rightarrow H K \cdot H D=H A \cdot H N$, do đó $A D N K$ nội tiếp.

  • Suy ra $\angle N H Q=\angle A H P=\angle H A P=\angle H D N$, suy ra $\angle H Q N=90^{\circ}$.

Bài 5. Cho đường tròn tâm $O$ và đường tròn tâm $I$ cắt nhau tại hai điểm $A$ và $B(O$ và $I$ nằm khác phía đối với đường thẳng $A B), I B$ cắt $(O)$ tại $E, O B$ cắt $(I)$ tại $F$. Qua $B$ vẽ đường thẳng $M N$ song song với $E F(M$ thuộc $(O), N$ thuộc $(I))$.

a) Chứng minh rằng $O A I E$ nội tiếp.

b) Chứng minh rằng: $A E+A F=M N$.

Lời giải.

a) Chứng minh rằng tứ giác $A O E F$ nội tiếp

Do hai đường tròn $(\mathrm{O})$ và $(\mathrm{I})$ cắt nhau tại $A$ và $B$ nên ta có: $A$ đối xứng với $B$ qua $O I$. Vậy: $\angle O A I=\angle O B I$

Ta có tam giác $\triangle O B E$ cân tại $O$ nên $\angle O B E=\angle O E B$, do $\angle O B E+\angle O B I=180^{\circ}$ nên $\angle O E B+\angle O B I=180^{\circ}$. Từ đó ta có: $\angle O E B+\angle O A I=180^{\circ}$

Vậy tứ giác $O A I E$ là tứ giác nội tiếp. Chứng minh tương tự ta có: tứ giác $O A I F$ là tứ giác nội tiếp.

$\angle O E A=\angle O I A$ (tứ giác $O A I E$ là tứ giác nội tiếp)

$\angle O I A=\angle O F A$ (tứ giác $O A I F$ là tứ giác nội tiếp)

Vậy: $\angle O E A=\angle O F A$ nên tứ giác $O A F E$ là tứ giác nội tiếp

b) Chứng minh rằng: $M N=A E+A F$

Bài toán cần chứng minh tương đương với: $A F=B N$ và $A E=B M$.

Ta chỉ cần chứng minh $A F=B N$ vì $A E=B M$ là điều tương tự.

Để chứng minh $A F=B N$. Ta chỉ cần chứng minh số đo cung $\mathrm{AF}$ bằng số đo cung $\mathrm{BN}(A F, B N$ lần lượt là dây căng cung $\mathrm{AF}$, cung $\mathrm{BN}$ trong đường tròn (I)). Hay chỉ cần chứng minh: số đo cung $\mathrm{AB}$ bằng số đo cung FN. Từ đó ta chứng minh: $\angle O F A=\angle F B N$ là bài toán được giải quyết.

Do $E F | M N$ nên ta có: $\angle O F E=\angle F B N$

Mà $\angle O F E=\angle O A E=\angle O E A=\angle O F A$ (tứ giác $A O E F$ là tứ giác nội tiếp)

Từ đó ta có: $\angle O F A=\angle F B N$ (đpcm)

Bài 6. Trên mặt phẳng cho 2013 điểm tùy ý sao cho trong ba điểm bất kì thì tồn tại 2 điểm mà khoảng cách giữa hai điểm đó luôn bé hơn 1. Chứng minh rằng tồn tại một đường tròn có bán kính bằng 1 chứa ít nhất 1007 điểm (kế cả biên).

Lời giải. Gọi $A$ là một điểm bất kì trong 2013 điểm trên. Lấy $A$ làm tâm vẽ đường tròn có bán kính bằng 1 .

Nếu 2012 điểm còn lại thuộc đường tròn $(A)$ thì bài toán được chứng minh xong. Giả tồn tại một số điểm nằm ngoài đường tròn tâm $(A)$. Lấy điểm $(B)$ bất kì trong các điểm đó và vẽ đường tròn tâm $(B)$ có bán kính bằng 1 .

Giả sử tồn tại một điểm $C$ nằm ngoài hai đường tròn $(A)$ và $(B)$ thì $A B, A C$ đều lớn hơn 1. Điều này vô lí.

Từ đó ta có tất cả các điểm đã cho đều thuộc trong hai đường tròn $(A)$ và $(B)$.

Theo nguyên lí Dirichlet sẽ tồn tại một đường tròn chứa $\frac{2012}{2}+1=1007$ điểm (đpcm).

 

 

 

 

 

 

 

 

 

 

 

 

 

 

 

 

 

 

 

 

 

 

 

 

 

 

 

 

 

 

 

 

 

 

 

 

 

 

 

 

 

 

 

 

 

 

 

 

 

 

 

ĐỀ THI VÀO LỚP 10 CHUYÊN TOÁN TRƯỜNG PHỔ THÔNG NĂNG KHIẾU 2012

Bài 1. (a) Giải hệ phương trình $\left\{\begin{array}{l}(x-y)^{2}=2 z-z^{2} \\ (y-z)^{2}=2 x-x^{2} \\ (z-x)^{2}=2 y-y^{2}\end{array}\right.$

(b) Cho hình vuông $A B C D$ cạnh $a$. $M$ và $N$ là hai điểm lần lượt nằm trên các cạnh $A B$ và $B C$ sao cho $\frac{A M}{A B}=\frac{C N}{C B}=x$ với $0<x<1$. Các đường thẳng qua $M, N$ song song với $B D$ lần lượt cắt $A D$ tại $Q$ và $C D$ tại $P$. Tính diện tích tứ giác $M N P Q$ theo $a$ và $x$ và tìm $x$ sao cho diện tích này lớn nhất.

Bài 2. Số nguyên dương $n$ được gọi là số điều hòa nếu như tổng các bình phương của các ước của nó ( kể cả 1 và $\mathrm{n}$ ) đúng bằng $(n+3)^{2}$.

(a) Chứng minh rằng số 287 là số điều hòa.

(b) Chứng minh rằng số $n=p^{3}$ ( $p$ nguyên tố ) không phải là số điều hòa.

(c) Chứng minh rằng nếu số $n=p q(p, q$ là các số nguyên tố khác nhau) là số điều hòa thì $n+2$ là số chính phương.

Bài 3. (a) Tìm tất cả các số thực $x$ thỏa $x^{2}-5 x+4+2 \sqrt{x-1} \geq 0$.

(b) Chứng minh rằng với các số không âm $a, b, c$ thỏa $a+b+c=3$ thì ta có bất đẳng thức $\sqrt{a}+\sqrt{b}+\sqrt{c} \geq a b+b c+a c$.

Bài 4. Cho tam giác $A B C$ vuông tại $A$. Trên đường thẳng vuông góc với $A B$ tại $B$ ta lấy điểm $D$ di động nằm cùng phía với $C$ đối với đường thẳng $A B$.

(a) Chứng minh rằng nếu $A C+B D<C D$ thì trên cạnh $A B$ tồn tại hai điểm $\mathrm{M}$ và $\mathrm{N}$ sao cho $\angle C M D=\angle C N D=90^{\circ}$

(b) Giả sử điều kiện trên được thỏa mãn. Đường thẳng qua $A$ song song với $M D$ cắt đường thẳng qua $B$ song song với $M C$ tại $E$. Chứng minh rằng đường thẳng $D E$ luôn đi qua một điểm cố định .

Bài 5. Cho đa giác đều n cạnh . Dùng 3 màu xanh, đỏ, vàng tô màu các đỉnh đa giác một cách tùy ý ( mỗi đỉnh được tô bởi một màu và tất cả các đỉnh đều được tô màu). Cho phép thực hiện thao tác sau đây : chọn hai đỉnh kề nhau bất kì ( nghĩa là hai đỉnh liên tiếp) khác màu và thay màu của hai đỉnh đó bằng màu còn lại.

(a) Chứng minh rằng bằng cách thực hiện thao tác trên một số lần ta luôn luôn làm cho các đỉnh của đa giác chỉ còn được tô bởi hai màu.

(b) Chứng minh rằng với $\mathrm{n}=4$ và $\mathrm{n}=8$, bằng cách thực hiện thao tác trên một.

số lần ta có thể làm cho các đỉnh của đa giác chỉ còn được tô bởi một màu.

LỜI GIẢI

Bài 1. (a) Giải hệ phương trình $\left\{\begin{array}{l}(x-y)^{2}=2 z-z^{2} \\ (y-z)^{2}=2 x-x^{2} \\ (z-x)^{2}=2 y-y^{2}\end{array}\right.$

(b) Cho hình vuông $A B C D$ cạnh $a$. $M$ và $N$ là hai điểm lần lượt nằm trên các cạnh $A B$ và $B C$ sao cho $\frac{A M}{A B}=\frac{C N}{C B}=x$ với $0<x<1$. Các đường thẳng qua $M, N$ song song với $B D$ lần lượt cắt $A D$ tại $Q$ và $C D$ tại $P$. Tính diện tích tứ giác $M N P Q$ theo $a$ và $x$ và tìm $x$ sao cho diện tích này lớn nhất.

Lời giải.

(a) Lấy (1) trừ (2) ta có:

$(x-2 y+z)(x-z)=x^{2}-z^{2}-2(x-z)=(x-z)(x+z-2) \Leftrightarrow 2(x-z)(y-1)=0 \Leftrightarrow x=z$ hoặc $y=1$.

  • Với $y=1$, ta có $(3) \Leftrightarrow(x-z)^{2}=1 \Leftrightarrow z=x+1, z=x-1$.

$+$ Với $z=x+1$, giải được $x=0, z=1$ và $x=1, z=2$. Khi đó ta có nghiệm $(0,1,1),(1,1,2)$.

$+$ Với $z=x-1$, giải ra được $x=1, z=0$ và $x=2, z=1$. Ta có nghiệm $(1,1,0)$ và $(2,1,1)$.

  • Với $x=z$, từ (3) ta có $y^{2}-2 y=0 \Leftrightarrow y=0, y=2$.

$+$ Với $y=0$ ta có $\left\{\begin{array}{l}x^{2}=2 z-z^{2} \\ z^{2}=2 x-x^{2}\end{array} \Leftrightarrow\left\{\begin{array}{l}2 z^{2}=2 z \\ x-z\end{array}\right.\right.$.

Giải được nghiệm $(0,0,0)$ và $(1,0,1)$.

$+$ Với $y=2$, giải ra được nghiệm $(1,2,1)$ và $(2,2,2)$.

Vậy hệ phương trình có 8 nghiệm.

(b) Chứng minh được $M N P Q$ là hình chữ nhật.

Ta có $\frac{M N}{A C}=\frac{M \dot{B}}{B A}=\frac{A B-A M}{A B}=1-\frac{A M}{A B}=1-x$ suy ra $M N=(1-x) a \sqrt{2}$.

$\frac{M Q}{B D}=\frac{A M}{A B}=x$, suy ra $M Q=x a \sqrt{2}$.

Từ đó $S=M N . M Q=2 a^{2} x(1-x)$ Mà $x(1-x) \leq \frac{1}{4}(x+1-x)^{2}=\frac{1}{4}$. Suy ra $S \leq \frac{a^{2}}{2}$. Đẳng thức xảy ra khi $x=\frac{1}{2}$.

Vậy diện tích đạt giá trị lớn nhất bằng $\frac{1}{2} a^{2}$ khi $M$ là trung điểm $A B$.

Bài 2. Số nguyên dương $n$ được gọi là số điều hòa nếu như tổng các bình phương của các ước của nó ( kể cả 1 và $\mathrm{n}$ ) đúng bằng $(n+3)^{2}$.

(a) Chứng minh rằng số 287 là số điều hòa.

(b) Chứng minh rằng số $n=p^{3}$ ( $p$ nguyên tố ) không phải là số điều hòa.

(c) Chứng minh rằng nếu số $n=p q(p, q$ là các số nguyên tố khác nhau) là số điều hòa thì $n+2$ là số chính phương.

Lời giải.

(a) Số $n=287$ có các ước dương là $1,7,41,287$. Ta có $1^{2}+7^{2}+41^{2}+287^{2}=$ $(287+3)^{2}$ nên 287 là số điều hòa.

(b) Các ước dương của $n=p^{3}$ là $1, p, p^{2}, p^{3}$. Giả sử $n$ là số điều hòa, ta có $(n+3)^{2}=1^{2}+p^{2}+p^{4}+p^{6} \Leftrightarrow p^{4}+p^{2}=6 p^{3}+8$. Suy ra $p \mid 8$ mà $p$ nguyên tố nên $p=2$. Thử lại thấy không thỏa, vậy $n=p^{3}$ không phải là số điều hòa với mọi số nguyên tố $p$.

(c) Các ước dương của $n=p q$ là $1, p, q, p q$. Vì $n$ là số điều hòa nên ta có: $1+p^{2}+q^{2}+p^{2} q^{2}=(p q+3)^{2} \Leftrightarrow p^{2}+q^{2}=6 p q+8 \Leftrightarrow(p+q)^{2}=$ $4(p q+2)$. Do 4 là số chính phương nên $p q+2$ cũng là số chính phương hay $n+2$ là số chính phương.

Bài 3. (a) Tìm tất cả các số thực $x$ thỏa $x^{2}-5 x+4+2 \sqrt{x-1} \geq 0$.

(b) Chứng minh rằng với các số không âm $a, b, c$ thỏa $a+b+C=3$ thì ta có bất đẳng thức $\sqrt{a}+\sqrt{b}+\sqrt{c} \geq a b+b c+a c$.

Lời giải.

(a) Điều kiện $x \geq 1$. Đặt $t=\sqrt{x-1}$. Khi đó $t \geq$ và $x=t^{2}+1$. Ta có bất phương trình:

$\left(t^{2}+1\right)^{2}-5\left(t^{2}+1\right)+4+2 t \geq \Leftrightarrow t^{4}-t^{2}+2 t \geq 0 \Leftrightarrow t(t+2)(t-1)^{2} \geq 0$

đúng với mọi $t \geq 0$.

Vậy nghiệm của bất phương trình là $x \geq 1$.

(b) Ta có $t^{2}-3 t+2 \sqrt{t}=\sqrt{t}(\sqrt{t}+2)(\sqrt{t}-1)^{2} \geq 0$. Suy ra $t^{2}+2 \sqrt{t} \geq 3 t$ với $\operatorname{mọi} t \geq 0$.

Áp dụng ta có $a^{2}+2 \sqrt{a} \geq 3 a, b^{2}+2 \sqrt{b} \geq 3 b, c^{2}+2 \sqrt{c} \geq 3 c$.

Suy ra $a^{2}+b^{2}+c^{2}+2(\sqrt{a}+\sqrt{b}+\sqrt{c}) \geq 3(a+b+c) \Leftrightarrow a^{2}+b^{2}+c^{2}+$ $2(\sqrt{a}+\sqrt{b}+\sqrt{c}) \geq(a+b+c)^{2} \Leftrightarrow \sqrt{a}+\sqrt{b}+\sqrt{c} \geq a b+b c+a c$ (đccm).

Bài 4. Cho tam giác $A B C$ vuông tại $A$. Trên đường thẳng vuông góc với $A B$ tại $B$ ta lấy điểm $D$ di động nằm cùng phía với $C$ đối với đường thẳng $A B$.

(a) Chứng minh rằng nếu $A C+B D<C D$ thì trên cạnh $A B$ tồn tại hai điểm $\mathrm{M}$ và $\mathrm{N}$ sao cho $\angle C M D=\angle C N D=90^{\circ}$

(b) Giả sử điều kiện trên được thỏa mãn. Đường thẳng qua $A$ song song với $M D$ cắt đường thẳng qua $B$ song song với $M C$ tại $E$. Chứng minh rằng đường thẳng $D E$ luôn đi qua một điểm cố định .

Lời giải.

(a) Xét đường tròn đường kính $C D$ có tâm $O$ là trung điểm $C D$. Gọi $I$ là trung điểm $A B$, khi đó $O I \perp A B$ và $O I$ là đường trung bình của hình thang $A C D B$ nên $O I=\frac{1}{2}(A C+B D)<\frac{C D}{2}$.

Do đó khoảng cách từ $O$ đến $A B$ nhỏ hơn bán kính đường tròn đường kính $C D$ nên $A B$ cắt đường tròn đường kính $A B$ tại hai điểm $M, N$. Suy ra $\angle C M D=\angle C N D=90^{\circ}$. Hơn nữa $\angle O C A+\angle O D B=180^{\circ}$ nên có một góc lớn hơn hoặc bằng $90^{\circ}$. Giả sử là $\angle A C D \geq 90^{\circ}$. Suy ra $O A>O C$. Suy ra $A$ nằm ngoài đường tròn $(O)$. Do đó $M, N$ thuộc đoạn $A B$.

(b) Gọi $E^{\prime}$ là giao điểm của đường thẳng qua $A$ song song với $M D$ với $C D$. Gọi $P$ là giao điểm của $M D$ với $A C, Q$ là giao điểm của $M C$ với $B D$. Theo định lý Thalet ta có: $\frac{C E^{\prime}}{C D}=\frac{C A}{C P}, \frac{C A}{C D}=\frac{B Q}{D Q}$. Suy ra $\frac{C E^{\prime}}{C D}=\frac{B Q}{D Q}$. Từ đó ta có $B E^{\prime}|| M C$. Suy ra $C, D, E$ thẳng hàng. Vậy đường thẳng $D E$ luôn qua điểm $C$ cố định.

Bài 5. Cho đa giác đều $n$ cạnh. Dùng 3 màu xanh, đỏ, vàng tô màu các đỉnh đa giác một cách tùy ý ( mỗi đỉnh được tô bởi một màu và tất cả các đỉnh đều được tô màu). Cho phép thực hiện thao tác sau đây : chọn hai đỉnh kề nhau bất kì ( nghĩa là hai đỉnh liên tiếp) khác màu và thay màu của hai đỉnh đó bằng màu còn lại.

(a) Chứng minh rằng bằng cách thực hiện thao tác trên một số lần ta luôn luôn làm cho các đỉnh của đa giác chỉ còn được tô bởi hai màu.

(b) Chứng minh rằng với $n=4$ và $n=8$, bằng cách thực hiện thao tác trên một số lần ta có thể làm cho các đỉnh của đa giác chỉ còn được tô bởi một màu.

Lời giải.

(a) Xét một dãy các đỉnh màu vàng $A V_{1} V_{2} \ldots V_{k} B$ (có thể chỉ gồm một đỉnh) được giới hạn bởi 2 đỉnh $A$ và $B$ (có thể trùng nhau) không phải màu vàng. Sử dụng thao tác đã cho ta đổi màu hai đỉnh $A$ và $V_{1}$ thành màu

thứ ba (hiển nhiên không phải màu vàng). Tiếp tục như thế đổi màu các đỉnh $\mid V_{2}, V_{3}, \ldots, V_{k}$ sang màu không phải vàng. Như vậy ta đã làm mất màu vàng trong dãy các đỉnh ở trên.

Bằng cách thực hiện như trên đối với dãy các điểm màu vàng khác ta suy ra có thể làm cho các đỉnh của đa giác chỉ còn được tô bởi hai màu xanh và đỏ.

(b) Do câu a) ta chỉ xét trường hợp các đỉnh của đa giác được tô bởi hai màu, chẳng hạn xanh và đỏ.

Bằng thao tác đã cho ta có hai kiểu chuyển màu bộ 4 đỉnh liên tiếp như sau:

$d d x x \rightarrow d v v x \rightarrow x x v x \rightarrow x d d x \rightarrow v v v v$ và $d x d x \rightarrow d v v d, d x x d \rightarrow v v v v$ (1)

Do tính đối xứng nên suy ra nếu một bộ 4 đỉnh mà trong đó có hai đỉnh cùng một màu và hai đỉnh còn lại cùng một màu khác thì ta chuyển cả 4 đỉnh về màu thứ ba.

Bằng cách dùng kiểu biến đổi trên ta có:

$d d d x \rightarrow d d v v \rightarrow x x x x$ (dùng (1)) và $d d x d \rightarrow d v v d \rightarrow x x x x(2)$.

Nghĩa là nếu có 3 đỉnh cùng màu, ta chuyển ta chuyển màu của 3 đỉnh đó về cùng màu của đỉnh thứ tư.

Như vậy bằng (1) và (2) ta có thể chuyển mày của mỗi bộ 4 đỉnh liên tiếp về cùng một màu. Điều này chứng minh cho trường hợp $n=4$.

Với $n=8$, ta chia 8 đỉnh thành 2 bộ 4 đỉnh. Như đã chứng minh ở trên, ta có thể làm cho mỗi bộ 4 đỉnh như thế có cùng màu. Nếu màu của hai bộ là như nhau thì ta có điều cần chứng minh. Nếu hai bộ khác nhau, chẳng hạn ta có kiểu tô màu $x x x x d d d d$. Ta có có phép biến đổi hai bộ liên tiếp: $x x x x d d d d \rightarrow x x x v v d d d \rightarrow x x x v \mid v d d d \rightarrow$ vvvvvvvvv(dùng (2)). Vậy ta đã chứng minh cho trường hợp $n=8$.

Đề và đáp án thi chọn đội dự tuyển lớp 10 năm 2012 – 2013

Bài 1. Cho $a,b$ là hai số thực thoả mãn $a+b\ge 0$. Chứng minh rằng:

$$\left(\dfrac{a^2+b^2}{2}\right)^3\ge 4(a^3+b^3)(ab-a-b).$$

Bài 2. Tìm tất cả các số nguyên dương $m,n$ để $\dfrac{5mn+5m}{3m^2+2n^2}$ là số nguyên.

Bài 3.  Cho tập hợp $X={1,2,\ldots,2n-1}$ gồm $2n-1$ số tự nhiên $(n\ge 2)$. Tô màu ít nhất $n-1$ phần tử của $X$ với điều kiện sau: nếu $a,b\in X$ (không nhất thiết phân biệt) được tô màu thì $a+b$ cũng được tô màu, miễn là $a+b\in X$. Gọi $S$ là tổng tất cả các số không được tô màu của $X$.

a/Chứng minh rằng $S\le n^2$.

b/Chỉ ra tất cả các phép tô màu sao cho $S=n^2$.

Bài 4. Cho đường tròn $(O)$ và dây cung $AB$ cố định khác đường kính. Gọi $C$ là điểm chính giữa cung lớn $AB$. Đường thẳng $d$ thay đổi qua $C$ cắt tiếp tuyến tại $A$ và tiếp tuyến tại $B$ của $(O)$ lần lượt tại $D,E$. Gọi $Q$ là giao điểm của $AE$ và $BD$. Chứng minh rằng đường thẳng $PQ$ luôn đi qua một điểm cố định khi $d$ thay đổi.

Giải

Bài 1. Ta xét các trường hợp sau:

  •  Nếu $ab<0$, ta có vế trái dương và vế phải âm nên bất đẳng thức đúng.
  •  Nếu $ab \ge 0$, kết hợp với $a+b \ge 0$, ta suy ra $a,b \ge 0.$

Áp dụng lần lượt các đánh giá $4xy \le (x+y)^2$ và $2xy \le x^2 + y^2$ thì:

$$\begin{align*} 4(a^3+b^3)(ab-a-b) & = 4(a+b)(ab-a-b)(a^2-ab+b^2) \\ & \leq a^2b^2(a^2-ab+b^2) \\ & \leq \dfrac{ab(a^2+b^2)^2}{4} \end{align*}$$

Mà ta có:

$$\dfrac{(a^2+b^2)^3}{8}=\dfrac{(a^2+b^2)^2}{4}.\dfrac{a^2+b^2}{2}\geq \dfrac{ab(a^2+b^2)^2}{4}.$$

Từ hai đánh giá trên, ta có đpcm.

Bài 2.

Đặt $k=\dfrac{5mn+5m}{3m^2 + 2n^2} \in \mathbb{N}^*$. Suy ra

$$3km^2 – 5(n+1)m + 2kn^2 = 0$$ là một phương trình theo ẩn $m$ với

$$\Delta = 25(n+1)^2 – 24k^2n^2 = (25-24k^2)n^2 + 50n + 25 \ge 0. (*)$$

Xét các trường hợp sau:

  • Nếu $k>1$, ta có:

$\Delta _1′ = 625 – 25\left( {25 – 24{k^2}} \right) = 600{k^2} > 0$, mà $25 – 24k^2 < 0$.

Suy ra bất phương trình $(*)$ có nghiệm khi $n \le \dfrac{25+10k\sqrt{6}}{24k^2-25}< 2$ (dễ dàng chứng minh).

Vì thế nên $n=1$ (do $n \in \mathbb{N^{*}}$). Ta có:

$$ \begin{aligned} k= \dfrac{10m}{3m^2 + 2} \in \mathbb{N^{*}} & \Rightarrow \dfrac{30m^2}{3m^2 + 2} \in \mathbb{N^{*}} \Rightarrow \dfrac{-20}{3m^2 + 2} \in \mathbb{N^{*}} \\ & \Rightarrow 3m^2 +2 \in \left\{ {2;5;10;20} \right\} \text{ vì } 3m^2+2 \ge 2, \forall m \\ & \Rightarrow m=1 \text{ do } m \in \mathbb{N^{*}}. \end{aligned} $$

Thử lại ta nhận $(m;n)=(1;1)$

  •  Nếu $k=1$ thì $\Delta = n^2 + 50n +25 = x^2$ ($x \in \mathbb{N}$) nên suy ra $$(n+x+25)(n-x+25) = 600.$$

Từ đây với lưu ý $n+x+25 > n-x+25 > 0, n+x+25 > 25$ ta có $$n \in \left\{ {126;52;28;10;6} \right\}.$$ Thay vào phương trình đầu, ta tìm được  $$(m;n)=(9;6),(5;10),(32;28),(32;52),(81;126).$$

Bài 3.

(a) Rõ ràng nếu $1$ được tô thì tất cả các số cũng sẽ được tô, kéo theo $S=0 \le n^2$, thỏa mãn. Do đó, ta chỉ cần xét $1$ không được tô. Gọi các số được tô là $$1 < a_1 < a_2 < \ldots < a_m \le 2n-1,$$

trong đó $m \ge n-1$. Ta sẽ chứng minh rằng với mọi $k$ mà $1 \le k \le m/2$ thì

$$a_k + a_{m-k+1} \ge 2n.$$

Giả sử ngược lại rằng $a_k+a_{m-k+1} <2n$ thì tổng hai số trên phải là số được tô màu. Do đó, nó phải thuộc tập hợp

$$Q = \left\{ {{a_{m – k + 2}};{a_{m – k + 3}};\ldots;{a_m}} \right\}.$$

Mặt khác lại xét chỉ số $i < k$ thì rõ ràng do dãy đang xét là tăng nên ta cũng có tổng ${a_i} + {a_{m – k + 1}}$ thuộc tập hợp $Q$ ở trên. Suy ra $|Q| \ge k,$ mâu thuẫn vì rõ ràng $Q$ chỉ có $k-1$ phần tử. Vì thế nên ta phải có $a_k + a_{m-k+1} \ge 2n.$

Đến đây, ta có ${a_k} + {a_{m – k + 1}} \ge 2n$ với mọi $k \in \left\{ {1;2;3;\ldots;m} \right\}$ nên

$$\sum\limits_{i = 1}^m {{a_i} = \frac{1}{2}} \sum\limits_{i = 1}^m {({a_i} + {a_{m – i + 1}}) \geqslant n(n – 1)}, \text{ suy ra }$$

$$S = \sum\limits_{i = 1}^{2n – 1} i – \sum\limits_{i = 1}^m {{a_i} \leqslant n(2n – 1) – n(n – 1) = {n^2}}.$$

(b) Để có $S=n^2$ thì dấu bằng xảy ra ở tất cả các đánh giá trên, tức là ta tô được đúng $m=n-1$ số và $a_k+a_{n-k}=2n$ với mọi $1 \le k \le n-1.$

Ta có $(2{{a}_{1}},{{a}_{1}}+{{a}_{2}},{{a}_{1}}+{{a}_{3}},\ldots ,{{a}_{1}}+{{a}_{n-2}})$ là một hoán vị của các số $({{a}_{2}},{{a}_{3}},\ldots ,{{a}_{n-1}}).$

Do tính tăng của hai dãy này nên ta có $$2{{a}_{1}}={{a}_{2}},{{a}_{1}}+{{a}_{2}}={{a}_{3}},\ldots ,{{a}_{1}}+{{a}_{n-2}}={{a}_{n-1}}.$$ Vì thế nên ${{a}_{k}}=k{{a}_{1}}$ với mọi $1\le k\le n-1.$ Mà $2n={{a}_{1}}+{{a}_{n-1}}=n{{a}_{1}}$ nên ta có ${{a}_{1}}=2,$ từ đây tìm được các tô duy nhất là $(2,4,6,\ldots ,2n-2)$ thỏa mãn đề bài.

Bài 4.

Giả sử $AD\cap BE=T,AB\cap DE=I$ và $TQ$ cắt $DE,AB$ lần lượt ở $X,S.$ Khi đó dễ thấy rằng

$(IX,DE)=(IS,AB)=-1.$

Mà $PI$ đi qua trung điểm cung lớn $AB$ của $(O)$ nên $PI$ là phân giác ngoài, kéo theo $PS$ là phân giác trong nên nó đi qua $N$ là trung điểm cung nhỏ $AB$ của $(O)$.

Gọi $M$ là trung điểm $AB.$ Theo tính chất phương tích thì $TN\cdot TC=T{{A}^{2}}=T{{B}^{2}}=TM\cdot TO$, mà $O$ là trung điểm $CN$ nên theo hệ thức Maclaurin thì $(TM,NC)=-1.$

Không có mô tả.

 

Lại có $(TQ,XS)=-1$ nên chùm $P(XS,QT)=-1$, mà $PX$ đi qua $C,$ $PS$ đi qua $N$ nên ta phải có $PQ$ đi qua $M$ là điểm cố định.

Nhận xét: Bài toán có thể xử lý theo hướng tự nhiên hơn bằng cách dùng định lý Ceva sin. Từ kết quả trên, ta còn thấy được rằng nếu lấy $CQ$ cắt $AB$ ở $K$ thì $PK$ là đối trung của tam giác $PAB,$ kéo theo $P,K,T$ thẳng hàng.

Đề thi và đáp án tuyển sinh vào lớp 10 PTNK không chuyên 2012

Đề thi vào lớp 10 trường Phổ thông Năng khiếu năm 2012

Bài 1. Cho phương trình $x^3 -4x\sqrt{x} +m + 1=0$ $(1)$

a) Giải phương trình $(1)$ khi $m=-33$

b) Tìm $m$ để phương trình $(1)$ có đúng hai nghiệm phân biệt $x_1$, $x_2$ thỏa $x_1^6 +x_2^6=82$.

Giải

Đặt $t=x\sqrt{x} \ge 0$.

a) Khi $m=-33$ ta có phương trình: $t^2 -4t -32=0$ có $2$ nghiệm $t=-4$, $t=8$, loại $t=-4$.

Với $t = 8$, thì $x = 4$

b) Ta có $\Delta’ =3-m >0 \Leftrightarrow m<3 $ và $\left\{ \begin{array}{l} S=t_1 + t_2 =4 \\ P=t_1t_2=m+1\end{array}\right. $

Khi đó $x_1^6 + x_2^6 = t_1^4 + t_2^4 = \left( t_1^2 + t_2^2 \right) ^2 – 2t_1^2 t_2^2 = 2m^2 -60m +194 $

$x_1^6 + x_2^6 =82 \Leftrightarrow m^2 -30m +56 =0 \Leftrightarrow \left[ \begin{array}{l} m=2 \,\, (n)\\\\ m=28 \,\, (l) \end{array} \right. $

Bài 2. Giải phương trình và hệ phương trình

a) $\sqrt{2x+7}-\sqrt{-3x-5}=1$.

b) $\left\{ \begin{array}{l} x^2 -2xy =1-2\sqrt{5}\\ xy-\dfrac{1}{10}y^2=\sqrt{5}-\dfrac{1}{2} \end{array} \right. $

Giải

a)Điều kiện: $-\dfrac{7}{2} \le x \le -\dfrac{5}{3}$

Phương trình tương đương:

$\sqrt{2x+7}=1+\sqrt{-3x-5}$

$\Leftrightarrow 5x+11 =2\sqrt{-3x-5} $

$\Leftrightarrow \left\{ \begin{array}{l} x\ge -\dfrac{11}{5} \\ 25x^2 +122x +141 =0 \end{array}\right.$

$\Leftrightarrow \left\{ \begin{array}{l} x\ge -\dfrac{11}{5} \\ \left[ \begin{array}{l} x=-3 \,\, (l) \\ x=-\dfrac{47}{25} \end{array}\right. \end{array} \right. $

b) Lấy $(1) + 2 \times (2)$, ta có phương trình $y^2 = 5x^2 \Leftrightarrow \left[ \begin{array}{l} y=x\sqrt{5} \\ y=-x\sqrt{5} \end{array}\right. $

Với $y=x\sqrt{5}$, thế vào $(1)$ ta có $x^2 – 2\sqrt{5}x^2 = 1-2\sqrt{5} \Leftrightarrow \left[ \begin{array}{l} x=1 \Rightarrow y=\sqrt{5} \\ x=-1 \Rightarrow y=-\sqrt{5} \end{array}\right. $

Với $y=-x\sqrt{5}$, thế vào $(1)$ ta có $x^2 + 2\sqrt{5}x^2 = 1-2\sqrt{5} \Leftrightarrow x^2 = \dfrac{1-2\sqrt{5}}{1+2\sqrt{5}}$ (VN)

Vậy nghiệm là: $\left( 1 ; \sqrt{5} \right)$, $\left( -1 ; -\sqrt{5} \right) $

Bài 3.

a) Rút gọn biểu thức: $$T = \left( \dfrac{2\sqrt{a}+ \sqrt{b}}{\sqrt{ab}+2\sqrt{a}-\sqrt{b}-2}-\dfrac{2-\sqrt{ab}}{\sqrt{ab}+2\sqrt{a}+\sqrt{b}+2} \right) $$ với $a,b \ge 0$ và $a \ne 1$.

Tìm giá trị lớn nhất của $T$ khi $a$ là số tự nhiên và $a \ne 1$.

b) Tìm $3$ số tự nhiên liên tiếp biết tổng $3$ tích của từng cặp số khác nhau của chúng là $1727$.

Giải

a) $MS1= \left( \sqrt{a}-1 \right) \left( \sqrt{b}+2 \right) $, $MS2= \left( \sqrt{a}+1 \right) \left( \sqrt{b}+2 \right) $

Quy đồng mẫu số chung $\left( \sqrt{a}-1 \right) \left( \sqrt{b}+2 \right) \left( \sqrt{a} +1 \right) = (a-1) \left( \sqrt{b} +2 \right) $ thì tử số bằng $(a+1)\left( \sqrt{b}+2 \right) $.

Suy ra $T= \dfrac{a+1}{a-1}$

$T= 1+ \dfrac{2}{a-1}$, $a=0 \Rightarrow T= -1$, $a>2 \Rightarrow T< 1+2 =3 =T (a=2) \Rightarrow T_{\max } =3$

b) Gọi $3$ số tự nhiên liên tiếp là $n – 1$, $n$ , $n + 1$ ($n \ge 1$), từ giả thiết ta có phương trình:

$(n-1)n+(n+1)n+(n-1)(n+1) =1727 \Leftrightarrow 3n^2 -1 = 1727 \Leftrightarrow n=24 \Rightarrow \text{ĐS}$

Bài 4. Tổng kết học kì $2$, trường trung học cơ sở $N$ có $60$ học sinh không đạt học sinh giỏi, trong đó có $6$ em từng đạt học sinh giỏi học kì $1$, số học sinh giỏi của học kì $2$ bằng $\dfrac{40}{37}$ số học sinh giỏi của học kì $1$ và có $8 \% $ số học sinh của trường không đạt học sinh giỏi học kì $1$ nhưng đạt học sinh giỏi học kì $2$. Tìm số học sinh giỏi học kì $2$ của trường biết rằng số học sinh của trường không thay đổi trong suốt năm học.

Giải

Gọi $x$ là số học sinh giỏi học kỳ $2$ của trường ($x$ nguyên dương).

Số học sinh của trường là $x + 60$ (học sinh)

Số học sinh giỏi của học kì $1$ là $\dfrac{37}{40}x$ (học sinh)

Ta có phương trình $\dfrac{8}{100}(x+60) -6= x-\dfrac{37}{40}x \Leftrightarrow x=240$.

Bài 5. Cho hình thang $ABCD$ ($AB // CD$) nội tiếp đường tròn $(C)$ tâm $O$, bán kính $R$ và có $\angle{DAB}=105^\circ$, $\angle{ACD}=30^\circ$.

a) Tính $\dfrac{DB}{DC}$ và tính $AB$ theo $R$.

b) Tiếp tuyến của $(C)$ tại $B$ cắt các đường thẳng $DO$, $DA$ lần lượt tại $M$, $N$. Tính $\dfrac{MN}{MD}$.

c) Gọi $E$ là trung điểm của $AB$, tia $DE$ cắt $MN$ tại $F$. Tính $\dfrac{BF}{BC}$.

Giải

a) Ta có $\angle{DAB}+ \angle{BCD}=180^\circ \Rightarrow \angle{BCD}=75^\circ $ $(1)$ $\Rightarrow \angle{ABC}= 105^\circ $

$\angle{ABD} =\angle{ACD}=30^\circ  \Rightarrow \angle{DBC} =\angle{ABC}-\angle{ABD}=105^\circ  -30^\circ  =75^\circ $ $(2)$

Từ $(1)$ và $(2)$ ta có $\angle{DBC} = \angle{DCB}$ ($=75^\circ $), nên $\triangle DCB$ cân tại $D$, suy ra $\dfrac{DB}{DC}=1$

Ta có $\angle{ACB}=75^\circ  -30^\circ  =45^\circ  \Rightarrow \angle{AOB}=2\angle{ACB} =90^\circ $, tam giác $AOB$ vuông cân tại $O$ nên $AB = AO\sqrt{2}=R\sqrt{2}$

b) Ta có $\angle{AOD}=2\angle{ACD}=60^\circ  \Rightarrow \Delta OAD$ đều $\Rightarrow \angle{ODA}=60^\circ $ hay $\angle{NDM}=60^\circ $

$\triangle DBC$ cân, nên $DO$ vừa là trung trực của $BC$ vừa là phân giác góc $\angle{BDC}$

$\angle{BOM}=180^\circ  -\angle{AOB} -\angle{AOD}=30^\circ  \Rightarrow \angle{OMB}= 60^\circ $ (do $OB \bot BM$)

Do đó tam giác $DMN$ đều, suy ra $\dfrac{MN}{MD} =1$

c) Gọi $E$ là trung điểm của $AB$, $\triangle AOB$ vuông cân tại $O$ nên $OE =AE$, $\angle{AEO}=90^\circ $

Ta có $\triangle ADE = \triangle ODE \Rightarrow \angle{AED} =\angle{OED}=45^\circ  , \angle{ADE}=\angle{ODE}=30^\circ$

$\Rightarrow DF$ là đường cao của tam giác $MDN$.

Gọi $I$ là trung điểm $BC$. Ta có $\angle{FDB}=15^\circ  =\angle{IDB}$

Khi đó $\triangle BFD = \triangle BID \Rightarrow BF =BI$ suy ra $\dfrac{BF}{BC}=\dfrac{1}{2}$

Đề thi và đáp án tuyển sinh vào lớp 10 TPHCM 2012

I. ĐỀ

Bài 1. Giải các phương trình và hệ phương trình sau:
a) $2x^2-x-3=0$
b) $ 2x-3y=7$ và $3x+2y=4 $
c) $x^4+x^2-12=0$
d) $x^2-2\sqrt{2}x-7=0$.

Bài 2.
a) Vẽ đồ thị $(P)$ của hàm số $y = \dfrac{1}{4}x^2$ và đường thẳng $(D): y =-\dfrac{1}{2}x + 2$ trên cùng một hệ trục tọa độ.
b) Tìm tọa độ giao điểm của $(P)$ và $(D)$ ở câu trên bằng phép tính.
Bài 3. Thu gọn các biểu thức sau:
a) $A = \dfrac{1}{{x + \sqrt x }} + \dfrac{{2\sqrt x }}{{x – 1}} – \dfrac{1}{{x – \sqrt x }}$ với $x > 0,x \ne 1$
b) $B = \left( {2 – \sqrt 3 } \right)\sqrt {26 + 15\sqrt 3 } – \left( {2 + \sqrt 3 } \right)\sqrt {26 – 15\sqrt 3 } $.
Bài 4. Cho phương trình $x^2-2mx+m-2=0$. ($x$ là ẩn số).

a) Chứng minh phương trình luôn có 2 nghiệm phân biệt với mọi $m$.
b) Gọi $x_1, x_2$ là các nghiệm của phương trình. Tìm $m$ để biểu thức $M = \dfrac{-24}{x_1^2+x_2^2-6x_1x_2}$ đạt giá trị nhỏ nhất.
Bài 5. Cho đường tròn $(O)$ có tâm $O$ và điểm $M$ nằm ngoài đường tròn $(O)$. Đường thẳng $MO$ cắt $(O)$ tại $E$ và $F$ (ME < MF). Vẽ cát tuyến MAB và tiếp tuyến $MC$ của $(O)$ ($C$ là tiếp điểm, $A$ nằm giữa hai điểm $M$ và $B$, $A$ và $C$ nằm khác phía đối với đường thẳng $MO$.
a) Chứng minh $MA.MB = ME.MF$.
b) Gọi $H$ là hình chiếu vuông góc của điểm $C$ lên đường thẳng $MO$. Chứng minh tứ giác $AHOB$ nội tiếp.
c) Trên nửa mặt phẳng bờ $OM$ có chứa điểm $A$, vẽ nửa đường tròn đường kính $MF$; nửa đường tròn này cắt tiếp tuyến tại $E$ của $(O)$ ở $K$. Gọi $S$ là giao điểm của hai đường thẳng $CO$ và $KF$. Chứng minh rằng đường thẳng $MS$ vuông góc với đường thẳng $KC$.
d) Gọi $P, Q$ lần lượt là tâm đường tròn ngoại tiếp các tam giác $EFS$ và $ABS$ và $T$ là trung điểm của $KS$. Chứng minh ba điểm $P, Q, T$ thẳng hàng.

II. ĐÁP ÁN

Bài 1.
a) $2x^2-x-3=0$ (a)
Vì phương trình (a) có $a-b+c=0$ nên
$(a) \Leftrightarrow x=-1$ hoặc $x=\dfrac{3}{2}$
b)
$2x-3y=7  (1)$  và
$3x+2y =4  (2)$
$\Leftrightarrow   2x-3y=7 (1)  và  $x+5y =-3  (3)  ((2)-(1))
$ \Leftrightarrow  -13y=13  ((1)-2(3))$  và  $x+5y=-3  (3)$
$\Leftrightarrow  y=-1$  và  $x=2$
c)  $x^4 + x^2 -12 =0$ $(c)$

Đặt $u= x^2 \ge 0$, phương trình trở thành: $u^2 + u -12 =0$ $(1)$
$(1)$ có $\Delta =49$ nên $(1) \Leftrightarrow u= \dfrac{-1+7}{2}=3$ hoặc $u=\dfrac{-1-7}{2}=-4$ (loại)
Do đó, $(c) \Leftrightarrow x^2=3 \Leftrightarrow x= \pm \sqrt{3}$
Cách khác:
$(c) \Leftrightarrow \left( x^2-3 \right) \left( x^2 +4 \right) =0 $

$\Leftrightarrow x^2 =3$

$\Leftrightarrow x = \pm \sqrt{3}$
d) $x^2 – 2\sqrt{2}-7=0$ (d)
$\Delta ‘ = 2+7=9$ do đó $(d) \Leftrightarrow x=\sqrt{2} \pm 3$.

Bài 2.
a) Đồ thị:

Lưu ý: $(P)$ đi qua $O(0;0)$, $( \pm 2 ;1)$, $(\pm 4; 4 )$
$(D)$ đi qua $(-4;4)$, $(2;1)$
b) Phương trình hoành độ giao điểm của $(P)$ và $(D)$ là:
$\dfrac{1}{4}x^2 = \dfrac{-1}{2}x+2 \Leftrightarrow x^2 +2x-8 =0 $

$\Leftrightarrow x=-4$ hoặc $x=2$
$y(-4) = 4,  y(2) =1$
Vậy tọa độ giao điểm của $(P)$ và $(D)$ là $(-4;4)$, $(2;1)$.
Bài 3.
a) $A= \dfrac{1}{x+\sqrt{x}}+ \dfrac{2\sqrt{x}}{x-1} – \dfrac{1}{x-\sqrt{x}}= \dfrac{x-\sqrt{x}-x-\sqrt{x}}{x^2-x}+ \dfrac{2\sqrt{x}}{x-1} $
$= \dfrac{-2\sqrt{x}}{x(x-1)}+\dfrac{2\sqrt{x}}{x-1}=\dfrac{2\sqrt{x}}{x-1} \left[- \dfrac{1}{x} +1 \right] = \dfrac{2\sqrt{x}(x-1)}{x(x-1)}=\dfrac{2}{\sqrt{x}}$ với $x>0$; $x\ne 1$
b) $B= \left( 2-\sqrt{3} \right) \sqrt{26+15\sqrt{3}}-\left( 2+\sqrt{3} \right) \sqrt{26-15\sqrt{3}}$
$= \dfrac{1}{\sqrt{2}}\left( 2-\sqrt{3} \right) \sqrt{52+30\sqrt{3}}-\dfrac{1}{\sqrt{2}}\left( 2+\sqrt{3} \right) \sqrt{52-30\sqrt{3}}$
$= \dfrac{1}{\sqrt{2}} \left( 2-\sqrt{3} \right) \sqrt{\left( 3\sqrt{3} + 5 \right)^2 } – \dfrac{1}{\sqrt{2}} \left( 2+\sqrt{3} \right) \sqrt{\left( 3\sqrt{3} – 5 \right)^2 }$
$= \dfrac{1}{\sqrt{2}} \left( 2-\sqrt{3} \right) \left( 3\sqrt{3} + 5 \right) – \dfrac{1}{\sqrt{2}} \left( 2+\sqrt{3} \right) \left( 3\sqrt{3} – 5 \right) $
$=\sqrt{2}$
Bài 4.

a) Phương trình (1) có:

$\Delta’ =m^2-m+2 = \left( m-\dfrac{1}{2} \right) ^2 + \dfrac{7}{4} >0 $ với mọi $m$ nên phương trình (1) có 2 nghiệm phân biệt với mọi $m$.
b) Do đó, theo Viet, với mọi $m$, ta có: $S=-\dfrac{b}{a} = 2m$; $P=\dfrac{c}{a}= m-2$
$M=\dfrac{-24}{\left( x_1+x_2 \right) ^2-8x_1x_2 } = \dfrac{-24}{4m^2-8m+16}= \dfrac{-6}{m^2-2m+4} = \dfrac{6}{(m-1)^2 + 3}$
Khi $m=1$ ta có $(m-1)^2 + 3$ nhỏ nhất
$\Rightarrow -M = \dfrac{6}{(m-1)^2+3}$ lớn nhất khi $m=1 $
$\Rightarrow M = \dfrac{-6}{(m-1)^2+3}$ nhỏ nhất khi $m=1$.
Vậy $M$ đạt giá trị nhỏ nhất là $-2$ khi $m=1$.
Bài 5.


a) Ta có $\angle MAE = \angle MFB$ (do $EFBA$ nội tiếp)
$\angle EMA = \angle BMF$
$\Rightarrow \triangle MEA \backsim \triangle MBF$
$\Rightarrow \dfrac{ME}{MB}= \dfrac{MA}{MF} \Rightarrow MA \cdot MB = ME \cdot MF $
b) Ta có $\triangle MCO$ vuông tại $C$, $CH$ là đường cao
$\Rightarrow MC^2 = MH \cdot MO$
$\triangle MAC \backsim \triangle MCB  (g-g) $
$\Rightarrow MC^2 = MA \cdot MB$
Do đó $MA \cdot MB = MH \cdot MO$
$\Rightarrow \dfrac{MA}{MO} = \dfrac{MH}{MB}$
mà $\angle AMH = \angle OMB $
$\Rightarrow \triangle AMH \backsim \triangle OMB $
$\Rightarrow \angle MAH = \angle MOB $
$\Rightarrow $ $AHOB$ nội tiếp
c) $\triangle MKF$ vuông tại $K$ có $KE$ là đường cao nên $MK^2 = ME \cdot MF$
Mà $MC^2 = MA \cdot MB = ME \cdot MF $
$\Rightarrow MK = MC$ (1)
Hai tam giác vuông $MKS$ và $MCS$ bằng nhau (cạnh huyền – cạnh góc vuông)
$\Rightarrow SK = SC$ (2)
Từ (1) và (2) $\Rightarrow$ $MS$ là trung trực của $KC$ $\Rightarrow MS \bot KC$
\item Gọi $I$ là giao điểm của $MS$ và $KC$
$\triangle MCS$ vuông tại $C$, $CI$ là đường cao nên $MC^2 = MI \cdot MS$
Mà $MC^2 = MA \cdot MB \Rightarrow MI \cdot MS = MA \cdot MB$
$\Rightarrow \dfrac{MA}{MS} = \dfrac{MI}{MB}$
$\angle AMI = \angle SMB \Rightarrow \triangle MAI \backsim \triangle MSB \Rightarrow \angle MIA = \angle MBS $
$\Rightarrow $ $ABSI$ nội tiếp (3)
$MI \cdot MS = MA \cdot MB = ME \cdot MF \Rightarrow \dfrac{ ME}{MS} = \dfrac{MI}{MF}$
Mà $\angle EMI = \angle SMF \Rightarrow \triangle MEI \backsim \triangle MSF $

$\Rightarrow \angle MEI = \angle MSF $
$\Rightarrow $ $EFSI$ nội tiếp (4)
Từ (3) và (4) suy ra hai đường tròn $(EFS)$ và $(ABS)$ cắt nhau tại $S$ và $I$
Mà $P$ và $Q$ là các tâm của hai đường tròn này
$\Rightarrow $ $PQ$ là trung trực của $SI$
$\triangle KIS$ vuông tại $I$ có $T$ là trung điểm của $KS$
$\Rightarrow TI = TS$
$\Rightarrow $ $T$ thuộc đường thẳng $PQ$.

 

Đáp án PTNK năm 2012

Bài 1. (Toán chung) Cho hình thang $ABC (AB||CD)$ nội tiếp đường tròn $(C)$ tâm $O$, bán kính $R$ và có $\angle DAB = 105^\circ, \angle ACD =30^\circ$.
a. Tính $\dfrac{DB}{DC}$ và tính $AB$ theo $R$.
b. Tiếp tuyến của $(C)$ tại $B$ cắt đường thẳng $DO$ và $DA$ lần lượt tại $M, N$. Tính $\dfrac{MN}{MD}$.
c. Gọi $E$ là trung điểm của $AB$, tía $DE$ cắt $MN$ tại $F$. Tính $\dfrac{BF}{BC}$.

Gợi ý

a.

  • Ta có $\angle DAB + \angle BCD = 180^\circ$, suy ra $\angle BCD = 75^\circ$ (1), suy ra $\angle ABC = 105^\circ$.
  • $\angle ABD = \angle ACD = 30^\circ$, suy ra $\angle DBC = \angle ABC – \angle ABD = 75^\circ$. (2)
  • Từ (1) và (2) ta có $\angle DBC = \angle DCB$, nên tam giác $DCB$ cân tại $D$, do đó $\dfrac{DB}{DC} = 1$.
  • Ta có $\angle ACB = 75^\circ – 30^\circ = 45^\circ$,suy ra $\angle AOB = 90^\circ$, tam giác $ABO$ vuông cân tại $O$ nên $AB = AO \sqrt{2} = R\sqrt{2}$.

b.

  • Ta có $\angle AOD = 2\angle ACD = 60^\circ$, suy ta tam giác $OAD$ đều. Suy ra $\angle ODA = 60^\circ$ hay $\angle NDM = 60^\circ$.
  • Tam giác $DBC$ cân, nên $DO$ cũng là trung trực của $BC$ và cũng là phân giác góc $\angle BDC$.
  • $\angle BOM = 180^\circ – \angle AOB – \angle AOD = 30^\circ$, suy ra $\angle OMB = 90^\circ – \angle BOM = 60^\circ$ (do $OB \bot BM$).
  • Do đó tam giác $DMN$ đều, suy ra $\dfrac{MN}{MD} = 1$.

c.

  • Gọi $E$ là trung điểm của $AB$, tam giác $AOB$ vuông cân tại $O$ nên $OE = AE, \angle AEO = 90^\circ$.
  • Ta có $\triangle ADE = \triangle ODE$ nên $\angle AED = \angle OED = 45^\circ$, $\angle ADE = \angle ODE = 30^\circ$, suy ra $DF$ là đường cao của tam giác $MDN$.
  • Gọi $I$ là trung điểm $BC$. Ta có $\angle FDB = 15^\circ = \angle IDB$.
  • Khi đó $\triangle BFD = \triangle BID$, suy ra $BF = BI$, suy ra $\dfrac{BF}{BC} = \dfrac{1}{2}$.

 

Bài 2. (Toán Chuyên) Cho hình vuông $ABCD$ cạnh $a$. $M$ và $N$ là hai điểm lần lượt nằm trên các cạnh $AB$ và $BC$ sao cho $\dfrac{AM}{AB} = \dfrac{CN}{CB} = x$ với $0 < x < 1$. Các đường thẳng qua $M , N$ song song với $BD$ lần lượt cắt $AD$ tại $Q$ và $CD$ tại $P$. Tính diện tích tứ giác $MNPQ$ theo $a$ và $x$ và tìm $x$ sao cho diện tích này lớn nhất.

 

Gợi ý
  • Chứng minh được $MNPQ$ là hình chữ nhật.
  • Ta có $\dfrac{MN}{AC} = \dfrac{MB}{BA} = \dfrac{AB-AM}{AB} = 1 – \dfrac{AM}{AB} = 1 – x$, suy ra $MN = (1-x)a\sqrt{2}$.
  • $\dfrac{MQ}{BD} = \dfrac{AM}{AB} = x$, suy ra $MQ = xa\sqrt{2}$.
  • Từ đó $S = MN.MQ = 2a^2x(1-x)$ Mà $x(1-x) \leq \dfrac{1}{4}(x+1-x)^2 = \dfrac{1}{4}$. Suy ra $S \leq \dfrac{a^2}{2}$. Đẳng thức xảy ra khi $x = \dfrac{1}{2}$.
  • Vậy diện tích đạt giá trị lớn nhất bằng $\dfrac{1}{2}a^2$ khi $M$ là trung điểm $AB$.

Bài 3 (Toán chuyên)  Cho tam giác $ABC$ vuông tại $A$. Trên đường thẳng vuông góc với $AB$ tại $B$ ta lấy điểm $D$ di động nằm cùng phía với $C$ đối với đường thẳng $AB$ .
a. Chứng minh rằng nếu $AC + BD < CD$ thì trên cạnh $AB$ tồn tại hai điểm $M$ và $N$ sao cho $\angle CMD =\angle CND = 90^\circ$
b. Giả sử điều kiện trên được thỏa mãn. Đường thẳng qua $A$ song song với $MD$ cắt đường thẳng qua $B$ song song với $MC$ tại $E$. Chứng minh rằng đường thẳng $DE$ luôn đi qua một điểm cố định . 

Gợi ý

a.

  • Xét đường tròn đường kính $CD$ có tâm $O$ là trung điểm $CD$. Gọi $I$ là trung điểm $AB$, khi đó $OI \bot AB$ và $OI$ là đường trung bình của hình thang $ACDB$ nên $OI = \dfrac{1}{2} (AC+BD) < \dfrac{CD}{2}$.
  • Do đó khoảng cách từ $O$ đến $AB$ nhỏ hơn bán kính đường tròn đường kính $CD$ nên $AB$ cắt đường tròn đường kính $AB$ tại hai điểm $M, N$. Suy ra $\angle CMD = \angle CND = 90^o$.
  • Hơn nữa $\angle OCA + \angle ODB = 180^o$ nên có một góc lớn hơn hoặc bằng $90^o$.
  • Giả sử là $\angle ACD \geq 90^o$. Suy ra $OA > OC$. Suy ra $A$ nằm ngoài đường tròn $(O)$. Do đó $M, N$ thuộc đoạn $AB$.

b.

  • Gọi $E’$ là giao điểm của đường thẳng qua $A$ song song với $MD$ với $CD$. Gọi $P$ là giao điểm của $MD$ với $AC$, $Q$ là giao điểm của $MC$ với $BD$.
  • Theo định lý Thalet ta có: $\dfrac{CE’}{CD} = \dfrac{CA}{CP}, \dfrac{CA}{CD} = \dfrac{BQ}{DQ}$. Suy ra $\dfrac{CE’}{CD} = \dfrac{BQ}{DQ}$.
  • Từ đó ta có $BE’ ||MC$. Suy ra $C, D, E$ thẳng hàng. Vậy đường thẳng $DE$ luôn qua điểm $C$ cố định.